dayna writes the integers 1,2,3,4,5,6,7,8,9,10,11,12 on a chalkboard, then she erases the integers from 1 through 6, as well as their multiplicative inverse $\mod{13}$. what is the only integer dayna does not erase?

Answers

Answer 1

The integers 1, 2, 3, 4, 5, 6, 7, 8, 9, 10, 11, 12 have been written on a chalkboard by Dayna. She then erased the integers from 1 through 6, as well as their multiplicative inverse $\mod{13}$.

We can find the multiplicative inverse of an integer { a modulo 13 } by using the extended Euclidean algorithm.The integers from 1 to 6 are 1, 2, 3, 4, 5, and 6.The multiplicative inverse of : 1 modulo 13 is 1, 2 modulo 13 is 7,  3 modulo 13 is 9, 4 modulo 13 is 10, 5 modulo 13 is 8, and 6 modulo 13 is 11.The only integer that Dayna does not erase is 12.

To learn more about Modulus & Integer : https://brainly.com/question/31132117

#SPJ11


Related Questions

I need help with this

Answers

sum area = -3x - 6y + 12 and product area = -36x - 72y.

what is rectangle?

A rectangle is a geometric shape that is defined as a four-sided flat shape with four right angles (90-degree angles) and opposite sides that are parallel and equal in length.

The area of a rectangle is given by the product of its length and width. Assuming that the length of the rectangle is given by -3x - 6y and its width is 12, we can express the area in terms of a sum and a product as follows:

Sum:

Area = length x width

Area = (-3x - 6y) + 12

Area = -3x - 6y + 12

Product:

Area = length x width

Area = (-3x - 6y) x 12

Area = -36x - 72y

Note that the product expression is not equal to the sum expression. This is because we used different assumptions for the length of the rectangle in each case.

Therefore, sum area = -3x - 6y + 12 and product area = -36x - 72y.

To learn more about rectange from the. given link:

https://brainly.com/question/29123947

#SPJ1

pls help Are the following lines parallel, perpendicular, or neither?

y = 2/3x − 4

y = −3/2x − 7

Responses


Parallel

Perpendicular

Neither

Answers

Answer:

Perpendicular.

Step-by-step explanation:

To determine whether the two lines are parallel, perpendicular, or neither, we need to compare their slopes.

The slope-intercept form of a line is y = mx + b, where m is the slope and b is the y-intercept. So we can rewrite the given equations in this form

y = 2/3x - 4 ==> slope = 2/3

y = -3/2x - 7 ==> slope = -3/2

Two lines are parallel if and only if their slopes are equal. Therefore, since the slopes of the two lines are different (2/3 and -3/2), they cannot be parallel.

Two lines are perpendicular if and only if their slopes are negative reciprocals of each other. That is, if the product of their slopes is -1. Therefore, we can check if the product of the slopes of the two lines is -1

(2/3) * (-3/2) = -1

Since the product of the slopes is -1, the two lines are perpendicular.

Therefore, the answer is: perpendicular.


Jackson owns a heating and cooling company. He is going to install a new furnace into a customer's house, but he must determine the volume of airflow
needed in order to select the best size of furnace. Find the volume of the house sketched below, where H1-10 ft., H2=9 feet, L=30 feet, and W=20 feet.
Volume of a rectangular prism is V=WLH (width x length x height) Volume of a triangular Prism is:
V =
a.ch
A
A target has a bull's-eye with a d X
O mathwarehouse.com
h (in this case a-9, c= 20, h= 30)
15

Answers

[tex]8700[/tex] cubic feet of airflow are required for the entire home. Jackson may utilize this information to determine the best furnace size for the customer's requirements.

Volume explain: What is it?

Volume is the quantity of space an object occupies, whereas capacity is a measurement of the substance—such as a solid, liquid, or gas—that an object can hold. While capacity may be measured in virtually any other unit, such as liters, gallons, pounds, etc., volume was determined in cubic units.

What are volume and what is its unit?

Volume, which is measured in cubic units, is the three dimensional space inhabited by material or surrounded by a surface. The cubic centimeter (m3), a derived unit, is the SI unit for volume.

Volume of the first section with height [tex]H1=10[/tex] ft:

[tex]V1 = WLH1 = (20 ft)(30 ft)(10 ft) = 6000[/tex] cubic feet

Volume of the second section with height [tex]H2=9[/tex] ft:

[tex]V2 = (1/2)WLH2 = (1/2)(20 ft)(30 ft)(9 ft) = 2700[/tex]cubic feet

Total volume of house,

[tex]V total = V1 + V2 = 6000[/tex] cubic feet + [tex]2700[/tex] cubic feet [tex]= 8700[/tex] cubic feet

Therefore, the volume of airflow needed for the house is [tex]8700[/tex] cubic feet.

To know more about volume visit:

https://brainly.com/question/1578538

#SPJ1

Subtract the given equation

3x-(4x-11)

Answers

Answer:

3x - (4x - 11) = 3x - 4x + 11 = -x + 11

Step-by-step explanation:

Determine whether the statement is true or false. If it is false, rewrite it as a true statement. A sampling distribution is normal only if the population is normal. Choose the correct answer below. A. The statement is true. B. The statement is false. A sampling distribution is normal only if n≥30. C. The statement is false. A sampling distribution is normal if either n≥30 or the population. D. The statement is false. A sampling distribution is never normal.

Answers

A sampling distribution is normal only if the population is normal. This  statement is false because A sampling distribution is normal only if n≥30.

If the underlying population is normally distributed, the sampling distribution (such as the sample mean distribution, also known as the xbar distribution) is also normally distributed. Even though the population is not normally distributed, the x(bar) distribution is approximately normal if n > 30, due to the central limit theorem. Some textbooks may use values ​​above 30, but after a certain threshold the x(bar) distribution is effectively "normal".

Option B is close, but misses the normal population part. n > 30 is not necessary if we know the population is normal.

A sampling distribution is the probability distribution of a statistic obtained from a large number of samples drawn from a particular population. The sampling distribution for a given population is the frequency distribution of a range of different outcomes that can occur in the population.

In statistics, a population is the entire basin from which a statistical sample is drawn. A population can refer to an entire population of people, objects, events, hospital visits, or measurements. Thus, a population can be said to be a global observation of subjects grouped by common characteristics.

Learn more about Population:

https://brainly.com/question/29117685

#SPJ4

what are the roots of 2x^2+10x+9=2x

Answers

The roots of the equation 2x² + 10x + 9 = 2x does not exist i.e no real roots

Calculating the roots of the equation

To find the roots of the given quadratic equation 2x² + 10x + 9 = 2x, we can start by rearranging the equation to the standard form of a quadratic equation

2x² + 10x + 9 - 2x = 0

Simplifying the left-hand side, we get:

2x² + 8x + 9 = 0

Now, we can use the quadratic formula to find the roots of the equation:

x = (-b ± √(b² - 4ac)) / 2a

where a = 2, b = 8, and c = 9.

Substituting these values into the formula, we get:

x = (-8 ± √(8² - 4(2)(9))) / 2(2)

Simplifying the expression under the square root, we get:

x = (-8 ± √-8) / 4

The square root of -8 is not a real number

So, the equation has no real root

Read more about quadratic equation at

https://brainly.com/question/24334139

#SPJ1

Terri pays a monthly cell phone fee of $10. She pays 5 cents for each minute that she talks. If Terri does not make any calls, what would her bill be?

Answers

Answer:

$10

Step-by-step explanation:

We Know

Terri pays a monthly cell phone fee of $10. She pays 5 cents for each minute that she talks.

Let C be the total cost, and x be the number of minutes she talks; we have the equation.

C = 0.05x + 10

If Terri does not make any calls, what would her bill be?

C = 0.05(0) + 10

C = $10

So, her bill will be $10

Suppose that a phone originally sold for $800 loses 3/5 of its value each year after it is released. After 2 years, how much is the phone worth?A. $800B. $1333C. $128D. $288​

Answers

The phone is worth $128 after 2 years, which is option C.

What is exponential decay ?

Exponential decay is a decrease in a quantity over time where the rate of decay is proportional to the current value. In this case, the value of the phone decreases by 3/5 each year after it is released. This means that the value after one year is 2/5 of the original value, and the value after two years is (2/5) times (2/5) of the original value. Exponential decay is a common phenomenon in many areas of science and mathematics, including radioactive decay, population growth and decay, and financial investments.

Calculating the worth of the phone :

The phone is not worth the same amount after 2 years, as it loses 3/5 of its value each year. We need to calculate its worth after 2 years.

Let's use the formula for exponential decay: [tex]A = A_0(1 - r)^t[/tex], where A is the final amount, [tex]A_0[/tex] is the initial amount, r is the decay rate, and t is the time elapsed.

In this case, the initial amount is $800, the decay rate is 3/5, and the time elapsed is 2 years. Substituting these values into the formula, we get:

[tex]A = 800(1 - 3/5)^2[/tex]

[tex]A = 800(2/5)^2[/tex]

[tex]A = 800(4/25)[/tex]

[tex]A = 128[/tex]

Therefore, the phone is worth $128 after 2 years, which is option C.

To know more about exponential decay visit :

brainly.com/question/2193799

#SPJ1

Question 9(Multiple Choice Worth 2 points)
(Irrational Numbers LC)
Describe in words where √63 would be plotted on a number line.
O Between 3 and 4, but closer to 3
O Between 3 and 4, but closer to 4
O Between 2 and 3, but closer to 2
O Between 2 and 3, but closer to 3

Answers

It would be plotted Between 3 and 4, but closer to 4

Answer:

Between 3 and 4, but closer to 4

PLEASE HURRY!!!!!!!!!!!!
Graph the solution to this inequality on the number line.

−5+x≥−3

Answers

Answer: 2

Step-by-step explanation:

To graph the solution to the inequality -5 + x ≥ -3 on the number line, we first need to isolate x.

Adding 5 to both sides of the inequality, we get:

x ≥ 2

This means that any value of x greater than or equal to 2 will satisfy the inequality. To graph this solution on a number line, we draw a closed circle at the point 2 and shade all the points to the right of 2, including the point 2 itself.

The resulting graph looks like this:

     ------•-------------------------------->

           2

The shaded region on the right of 2 represents all the values of x that make the inequality true.

find the percent of the discount: a $30 board game on sale for 21​

Answers

well, we know the discount is just 30 - 21 = 9, so hmm if we take 30(origin amount) to be the 100%, what's 9 off of it in percentage?

[tex]\begin{array}{ccll} Amount&\%\\ \cline{1-2} 30 & 100\\ 9& x \end{array} \implies \cfrac{30}{9}~~=~~\cfrac{100}{x} \\\\\\ 30x=900\implies x=\cfrac{900}{30}\implies x=30[/tex]

The store sells a television for $1000. customers can choose to receive 10% discount and pay it off at a simple interest rate of 4% or they can choose to pay the full price and pay it off in 3 years with no interest. which option is better

Answers

Option 1 with the discount and 4% simple interest has a total cost of $972, while Option 2 with no discount and no interest has a total cost of $1000. Option 1 is the better choice as it has a lower total cost.

What is simple interest?

Simple interest is a type of interest that is calculated on the original principal amount of a loan or investment. It is a fixed percentage of the principal amount that is paid by the borrower or earned by the lender over a specific period of time.

According to question:

To compare the two options, we need to calculate the total cost of each option and compare them.

Option 1: 10% discount and pay off with 4% simple interest

The discount reduces the price of the television to $1000 x 0.9 = $900. If the customer chooses to pay it off at 4% simple interest, the total cost would be:

Total cost = $900 + ($900 x 0.04 x 3) = $972

Option 2: Full price and pay off in 3 years with no interest

The total cost of this option would be simply the full price of $1000 paid over 3 years, so:

Total cost = $1000 / 3 = $333.33 per year x 3 years = $1000

Comparing the two options, we see that Option 1 with the discount and 4% simple interest has a total cost of $972, while Option 2 with no discount and no interest has a total cost of $1000. Therefore, Option 1 is the better choice as it has a lower total cost.

To know more about simple interest visit:

https://brainly.com/question/29210957

#SPJ1

The complete question is: The store sells a television for $1000. customers can choose to receive 10% discount and pay it off at a simple interest rate of 4% or they can choose to pay the full price and pay it off in 3 years with no interest. which option is better?

Option 1 with the discount and 4% simple interest.

Option 2 with no discount and no interest.

Can anyone solve this problem please? Thanks!

Answers

The trapezoid has a surface area of 480 square units.

What is the measurement for a trapezoid's area?

So, a trapezoid measured in feet offers an area in square feet; one measured in millimetres gives an area in square centimetres; and so on. If it's simpler for you, you can add the lengths of the bases and then divide the total by two. Keep in mind that multiplication by 12 is equivalent to dividing by 2.

We must apply the formula for a trapezoid's area to this issue in order to find a solution:

[tex]A = (1/2) * (a + b) * h[/tex]

where h is the trapezoid's height (or altitude) and a and b are the lengths of its parallel sides.

The values for a, b, and h are provided to us, allowing us to change them in the formula:

A = (1/2) * (20 + 60) * 12

A = (1/2) * 80 * 12

A = 480 square units

To know more about trapezoid visit:-
https://brainly.com/question/8643562

#SPJ1

Halla los números desconocidos de estas operaciones
A)872+. +173=2000
B)9180:. =102
C). -99=706
Con los mismos números y las mismas operaciones podemos obtener diferentes resultados,coloca los paréntesis de manera que se obtengan los resultados indicados. A)3+5x7-2=40
B)3+5×7-2=54
C)3+5×7-2=28
ES PARA HOY PORFAVOR☹,PUEDEN HACER EN UNA HOJA O ESCRIBIR ASI PERO EXPLIQUEN BIEN!!!!!!AYUDA SI NO SABEN NO RESPONDAD

Answers

In equation A the missing number is 955, In equation B the missing number is 90 and In equation C the missing number is 805.

A) To find the missing number in the equation 872 + ? + 173 = 2000, we need to subtract 872 and 173 from 2000, which gives us:

2000 - 872 - 173 = 955

Therefore, the missing number is 955.

B) To find the missing number in the equation 9180 ÷ ? = 102, we need to divide 9180 by 102, which gives us:

9180 ÷ 102 = 90

Therefore, the missing number is 90.

C) To find the missing number in the equation ? - 99 = 706, we need to add 99 to 706, which gives us:

706 + 99 = 805

Therefore, the missing number is 805.

To obtain the indicated results with the same numbers and operations, we need to use parentheses to change the order of operations.

A) 3 + (5x7) - 2 = 40

B) (3 + 5) × 7 - 2 = 54

C) 3 + (5 × (7-2)) = 28

Equations are used extensively in various fields of science, engineering, economics, and finance, to name a few. It is formed by placing an equal sign between the two expressions. Equations are used to solve problems and find unknown values.

An equation can contain variables, constants, and mathematical operations such as addition, subtraction, multiplication, and division. The variables in an equation represent unknown values that need to be found, while the constants are known values that are already given. Solving an equation involves manipulating the expressions on both sides of the equal sign using mathematical operations to isolate the variable on one side and constants on the other. The final solution obtained is the value of the variable that satisfies the equation..

To learn more about Equation visit here:

brainly.com/question/29538993

#SPJ4

Complete Question: -

Find unknown numbers of these operations

A ) 872 +. + 173 = 2000

B ) 9180:. = 102

C ). -99 = 706

With the same numbers and the same operations we can obtain different results, place the parentheses so that the indicated results are obtained.

A ) 3 + 5 x 7-2 = 40

B ) 3 + 5 × 7-2 = 54

C ) 3 + 5 × 7-2 = 28

IT'S FOR TODAY PLEASE ☹, CAN DO IN A LEAF OR WRITE ASI BUT EXPLAIN WELL!!!!!!HELP IF THEY DON'T KNOW NO RESPOND

If all other factors are held constant, which of the following results in an increase in the probability of a Type II error? a. The true parameter is farther from the value of the null hypothesis. b. The sample size is increased. c. The significance level is decreased d. The standard error is decreased. e. The probability of a Type II error cannot be increased, only decreased

Answers

If all other factors are held constant, then the true parameter is farther from the value of the null hypothesis which is an increase in the probability of a Type II error.The correct option is A.

The true parameter is farther from the value of the null hypothesis.

When the true parameter is farther away from the value of the null hypothesis, it increases the probability of a Type II error. This is because the null hypothesis will have a harder time rejecting the true parameter.

The other factors - increasing sample size, decreasing significance level, and decreasing standard error - all result in a decreased probability of a Type II error.

To learn more about the null hypothesis:

https://brainly.com/question/15980493

#SPJ11

Given the triangle, find the length of X. Give your answer in simpliest radical form.

Answers

Answer:

x = 4[tex]\sqrt{2}[/tex]

Step-by-step explanation:

using the cosine ratio in the lower right triangle and the exact value

cos45° = [tex]\frac{1}{\sqrt{2} } }[/tex] , then

cos45° = [tex]\frac{adjacent}{hypotenuse}[/tex] = [tex]\frac{4}{x}[/tex] = [tex]\frac{1}{\sqrt{2} }[/tex]  ( cross- multiply )

x = 4[tex]\sqrt{2}[/tex]

why can't we use mean when a data set has one or two values that are much higher than all of the others

Answers

The reason we can't use the mean when a data set has one or two values that are much higher than all of the others is that it skews the average, making it not representative of the rest of the data.

What is the mean?

The mean is a numerical measure of the central tendency of a data set. It is calculated by dividing the sum of all the values in a data set by the number of data points.

A data set is a collection of observations or measurements that are analyzed to obtain information. It can be represented graphically, in tabular form, or in any other format. The data set may be a sample or the entire population.

If a data set has one or two extremely high or low values, it can significantly impact the mean. These values are known as outliers. The outliers can cause the mean to be higher or lower than the actual middle value of the data.

Hence, in such cases, the median is a better choice for finding the central tendency of the data. The median is the middle value of the data set, and it is less affected by outliers than the mean. The mode, which is the value that occurs most frequently in the data set, is also a measure of central tendency that is less sensitive to outliers than the mean.

Learn more about mean here:

https://brainly.com/question/31101410

#SPJ11

Help due soon !!!!!!!!!

Answers

An expression for the length of the rectangle in terms of A is [tex]$\boxed{L=x+5}$[/tex]

How to find the expression?

We are given that the area of a rectangle is [tex]$A=x^2+x-15$[/tex], and we want to find an expression for the length of the rectangle in terms of A.

Recall that the area of a rectangle is given by the formula: [tex]$A=L\cdot W$[/tex], where L is the length and W is the width. We can use this formula to write L in terms of A and W as [tex]$L=\frac{A}{W}$[/tex].

We know that the rectangle has a length and a width, so we need to find an expression for the width W in terms of A. We can rearrange the given formula for A to solve for W:

[tex]&& \text{(substitute }L=x+5\text{)}[/tex]

[tex]W&=\frac{x^2+x-15}{x+5} && \text{(divide both sides by }x+5\text{)}[/tex]

Now that we have an expression for W in terms of A, we can substitute it into our expression for L to get:

[tex]L&=\frac{A}{W}[/tex]

[tex]&=\frac{x^2+x-15}{\frac{x^2+x-15}{x+5}} && \text{(substitute the expression we found for }W\text{)}\&=x+5[/tex]

Therefore, an expression for the length of the rectangle in terms of A is [tex]$\boxed{L=x+5}$[/tex]

To know more about rectangle visit:

https://brainly.com/question/29123947

#SPJ1

g in acid base reactions, the hydrogen ions from the acid and the hydroxide ions from the base neutralize each other. khp has one ionizable hydrogen; this means that one mole of sodium hydroxide neutralizes one mole of khp. from experiment 1, calculate the exact molarity of the sodium hydroxide. (hint: use the mass of khp and do a stoichiometry problem.....) tip: khp is not the chemical formula. khp stands

Answers

In the following question, among the conditions given, the statement is said to be, the exact molarity of the NaOH solution is 0.0960 M.

The question is asking to calculate the exact molarity of the sodium hydroxide from Experiment 1.
KHP stands for potassium hydrogen phthalate, and one mole of sodium hydroxide (NaOH) will neutralize one mole of KHP. To solve the problem, use the mass of KHP and a stoichiometry problem.
First, calculate the number of moles of KHP:
Moles KHP = (Mass KHP (g) / Molar Mass KHP (g/mol))
Then, calculate the moles of NaOH:
Moles NaOH = (Moles KHP * Mole Ratio NaOH/KHP)
Finally, calculate the molarity of NaOH:
Molarity NaOH = (Moles NaOH / Volume NaOH (L))

For more such questions on NaOH

https://brainly.com/question/15973235

#SPJ11

There is a 0.99962 probability that a randomly selected 28​-year-old female lives through the year. An insurance company wants to offer her a​ one-year policy with a death benefit of ​$500,000. How much should the company charge for this policy if it wants an expected return of ​$400 from all similar​ policies?

Answers

In order to expect a return on $400 from across all policies of a similar nature, the insurance firm should charge the policy for about $501.88.

How then do we return a value?

Return[expr] leaves control structures that are present during a function's definition and returns the value expression for the entire function. Even if it comes inside other functions, yield takes effect as quickly as it is evaluated. Functions like Scan can use Return inside of them.

Since p is the chance that the 28-year-old woman survives the year and is given as 0.99962, we can enter this number into the equation for n as follows: n = 400(0.99962)/500,400 n 0.799

In light of this, the insurance provider should impose a premium of: Premium = 400/n

$501.88 is the premium ($Premium = 400/0.799)

To know more about return visit:

https://brainly.com/question/28562900

#SPJ1

Analyze the proportion below and complete the instructions that follow. Use a model to find the missing value in the proportion. A. 4 B. 5 C. 10 D. 22 Please select the best answer from the choices provided A B C D

Answers

Step-by-step explanation:

The area of a rectangle is 1,872 ft2. The ratio of the length to the width is 9:13. Find the perimeter of the rectangle.

176 ft

You want to make a scale drawing of your bedroom to help arrange your furniture. You decide on a scale of 3 in. = 2 ft. Your bedroom is a 12 ft by 14 ft rectangle. What should the dimensions of your drawing be?

18 in. by 21 in.

If 5/y + 7/x=24 and 12/y + 2/x=24, find the ratio of x to y.

5/7

Simplify the ratio 8ft/12in. Use the conversion 12 in. = 1 ft.

8/1

Analyze the proportion below and complete the instructions that follow.

2x+5/3 = x-5/4

-7

If a+b/2a-b = 5/4 and b/a+9 = 5/9, find the value of b.

30

Analyze the ratio below and complete the instructions that follow.

$30:$6

Simplify the ratio.

5:1

If 14/3 = x/y then 14/x =

3/y

Analyze the diagram below and complete the instructions that follow.

In the diagram, AB:BC is 3:4 and AC = 42. Find BC.

24

Analyze the diagram below and complete the instructions that follow.

If AB:BC is 3:11, solve for x.

9

If a, b, c, and d are four different numbers and the proportion a/b = c/d is true, which of the following is false?

b/a = c/d

Analyze the diagram below and complete the instructions that follow.

Find the ratio of the width to the length of the rectangle, then simplify the ratio. Use the conversion 100 cm = 1 m.

3/4

Simplify the ratio 3 gal./24 qt. Use the conversion 4 qt = 1 gal.

1/2

The area of a rectangle is 4,320 ft2. The ratio of the length to the width is 6:5. Find the length of the rectangle.

72 ft

Analyze the diagram below and complete the instructions that follow.

Given that CB/CA = DE/DF, find BA.

10.5

Analyze the proportion below and complete the instructions that follow.

2/3 = 8/x

3, 8

Analyze the diagram below and complete the instructions that follow.

Are the polygons shown here similar? Justify your answer. The images are not drawn to scale.

Yes, PQR ~TSV with a scale factor of 1:√3

All __________ are similar.

squares

Analyze the diagram below and complete the instructions that follow.

Determine which 2 triangles are similar to each other. The images are not drawn to scale.

GHI ~ JKL

Analyze the diagram below and complete the instructions that follow.

Pentagon PQRST ~ pentagon XYZVW. Find the value of b. The images are not drawn to scale.

3

Analyze the diagram below and complete the instructions that follow.

If ABC ~ XYZ, find XY. The images are not drawn to scale.

24

ABC is a right triangle. The legs of ABC are 9 ft and 12 ft. The shortest side of XYZ is 13.5 ft, and ABC ~ XYZ How long is the hypotenuse of XYZ?

22.5 ft

PLEASE HELP!!! WILL MARK BRANLIEST!!!

Answers

Answer:

The point z = 3+4i is plotted as a blue dot, and the two square roots are plotted as a red dot and a green dot. The magnitudes of z and its square roots are shown by the radii of the circles centered at the origin.

Step-by-step explanation:

qrt(z) = +/- sqrt(r) * [cos(theta/2) + i sin(theta/2)]

where r = |z| = magnitude of z and theta = arg(z) = argument of z.

Calculate the magnitude of z:

|r| = sqrt((3)^2 + (4)^2) = 5

And the argument of z:

theta = arctan(4/3) = 0.93 radians

Now, find the two square roots of z:

sqrt(z) = +/- sqrt(5) * [cos(0.93/2) + i sin(0.93/2)]

= +/- 1.58 * [cos(0.47) + i sin(0.47)]

= +/- 1.58 * [0.89 + i*0.46]

Using a calculator, simplify this expression to:

sqrt(z) = +/- 1.41 + i1.41 or +/- 0.2 + i2.8

The population of a slowly growing bacterial colony after t hours is given by p(t)=3t^2+24t+200. Find the growth rate after 2 hours.

Answers

The growth rate of a bacterial colony after a 2 hours  is given by the derivative of its population function with respect to time is 36 .

The growth rate of a bacterial colony is given by the derivative of its population function.

Thus, we need to find the derivative of the population function p(t) with respect to time t, and then evaluate it at t = 2 to get the growth rate after 2 hours.

p(t) = 3t² + 24t + 200

Taking the derivative of p(t) with respect to t, we get:

p'(t) = 6t + 24

Now, evaluating p'(t) at t = 2, we get:

p'(2) = 6(2) + 24 = 36

Therefore, the growth rate of the bacterial colony after 2 hours is 36.

To practice more questions about the growth rate:

https://brainly.com/question/25849702

#SPJ11

LMN is a straight angle. Find m LMP and m NMP​

Answers

From the given information provided, the value of angle LMP and angle NMP is 77 and 103 degrees respectively.

Since LMN is a straight angle, it measures 180 degrees.

We are given the measures of LMP and NMP, and we are told that LMP + NMP = LMN. Therefore, we can set up an equation:

LMP + NMP = LMN

(-16x + 13) + (-20x + 23) = 180

Simplifying and solving for x, we get:

-36x + 36 = 180

-36x = 144

x = -4

Now that we have found the value of x, we can substitute it back into the expressions for LMP and NMP to find their measures:

LMP = -16x + 13 = -16(-4) + 13 = 77 degrees

NMP = -20x + 23 = -20(-4) + 23 = 103 degrees

Therefore, the measures of LMP and NMP are 77 degrees and 103 degrees, respectively, and the measure of LMN is 180 degrees.

Question - LMN is a straight angle. LMP = -16x + 13 NMP =  -20x + 23 LMP + NMP = LMN What are the measures?

Learn more about angle here: brainly.com/question/25770607

#SPJ4

Sophie invested $92,000 in an account paying an interest rate of 6 1/8% compounded

continuously. Damian invested $92,000 in an account paying an interest rate of 6 5/8%

compounded monthly. After 14 years, how much more money would Damian have in

his account than Sophie, to the nearest dollar?

Answers

Answer:

Step-by-step explanation:

To solve this problem, we need to use the formula for compound interest:

A = P*e^(rt)

where A is the final amount, P is the principal (initial investment), e is the base of the natural logarithm (approximately 2.71828), r is the interest rate (expressed as a decimal), and t is the time (in years).

For Sophie's account, we have:

P = $92,000

r = 6 1/8% = 0.06125 (as a decimal)

t = 14 years

A = 92000*e^(0.06125*14)

A = $219,499.70 (rounded to the nearest cent)

For Damian's account, we have:

P = $92,000

r = 6 5/8% = 0.06625/12 = 0.005521 (as a monthly decimal rate)

t = 14*12 = 168 months

A = 92000*(1+0.005521)^168

A = $288,947.46 (rounded to the nearest cent)

Now we can subtract Sophie's final amount from Damian's final amount to find the difference:

Difference = $288,947.46 - $219,499.70

Difference = $69,447.76

Therefore, Damian would have about $69,448 more in his account than Sophie, to the nearest dollar.

Determine the force in each member of the truss with the method of joints and state if the

members are in tension (T) or compression (C). Set d = 1 m and P = 10 kN. (Hint: Look for zero-

force members to simplify the calculations)

Answers

if the members are in tension or compression. Identify all zero force members: Likewise, we can find the reaction force at A by taking minutes about point A: RA x 8m - 5kN x 8m - 5kN x 8m = 0 RA = 5kN

To begin with, we really want to find the reaction forces at An and G.

We can do this by taking minutes about point G.

We realize that the amount of minutes at any point is zero when the framework is in equilibrium.

Consequently, we can compose: 5kN x 8m - RA x 10m = 0 RA = 4kN

Likewise, we can find the reaction force at A by taking minutes about point A: RA x 8m - 5kN x 8m - 5kN x 8m = 0 RA = 5kN

Since we have two distinct qualities for RA, we can presume that the framework isn't in equilibrium.

This really intends that there should be some outside force following up on the framework.

The two obscure forces are at first thought to be ductile (for example pulling away from the joint). In the event that this underlying supposition is mistaken, the registered upsides of the pivotal forces will be negative, meaning pressure.

to know more about forces click here:

https://brainly.com/question/30284058

#SPJ4

the complete question is:

Question l Find the forces in members HE; FH, FE; and FC of the truss as shown in Figure Q1. State if the members are in tension or compression. Identify all zero force members: (10 marks) 8 m 5 KN 8 m 8 m 5 KN 8 m 10 m Figure Q1.

find the values of a and b such that
x^2- +5=(x-a)^2+b

Answers

The value οf a = 1/2  and b = 19/4 in the equatiοn x² - x + 5 = (x-a)² + b.

What dο yοu mean by algebra?

The part οf mathematics in which letters and οther general symbοls are used tο represent numbers and quantities in fοrmulae and equatiοn is called algebra.

x² - x + 5 = (x-a)² + b

x² - x + 5 = x² + a² - 2xa + b

-x + 5 = -2xa + a² + b

By matching cοrrespοnding terms,

2a = 1          and        a²+ b = 5

a= 1/2          and        a²+ b = 5

Substituting value οf "a"

(1/2)² + b = 5

1/4 + b = 5

b = 5- 1/4

b = 19/4

Thus, a = 1/2  and b = 19/4.

To learn more about algebra, click on the link given below

https://brainly.com/question/22399890

#SPJ1

A local winery wants to create better marketing campaigns for its white wines by understanding its customers better. One of the general beliefs has been that higher proportion of women prefer white wine as compared to men. The company has conducted a research study in its local winery on white wine preference. Of a sample of 400 men, 120 preferred white wine and of a sample of 500 women, 170 preferred white wine. Using a 0.05 level of significance, test this claim.INPUT Statistics required for computation170 = Count of events in sample 1500 = sample 1 size120 = Count of events in Sample 2400 = sample 2 size0.05 = level of significance0 = hypothesized differenceOUTPUT Output valuesSample 1 Proportion 34.00%Sample 2 Proportion 30.00%Proportion Difference 4.00%Z α/2 (One-Tail) 1.645Z α/2 (Two-Tail) 1.960Standard Error 0.031Hypothesized Difference 0.000One-Tail (H0: p1 − p2 ≥ 0)Test Statistics (Z-Test) 1.282p-Value 0.900One-Tail (H0: p1 − p2 ≤ 0)Test Statistics (Z-Test) 1.282p-Value 0.100Two-Tail (H0: p1 − p2 = 0)Test Statistics (Z-Test) 1.276p-Value 0.202Group of answer choicesThis is a one-tail test and the data does support the claim that higher proportion of women prefer white wine as compared to men.This is a one-tail test and the data does not support the claim that higher proportion of women prefer white wine as compared to men.This is a two-tail test and the data does support the claim that higher proportion of women prefer white wine as compared to men.This is a two-tail test and the data does not support the claim that higher proportion of women prefer white wine as compared to men.Question 2. Based on the study results presented in the last question, what is the upper bound for the proportion differences between women and men for a 95% confidence interval?(Note: Please enter a value with 4 digits after the decimal point. For example, if you computed an upper boundary of 23.456% or .23456, you would enter it here in decimal notation and round it to four digits, thus entering .2346).

Answers

Answer:

235.65

Step-by-step explanation:

Describe the error in finding the distance between A(6, 2) and B(1,−4)

Answers

The error is the substitution of coordinates. Coordinates are ordered pairs of points that help us locate any point in a 2D plane or 3D space.

Cartesian coordinates, also known as the coordinates of a point in a 2D plane, are two integers, or occasionally a letter and a number, that identifies a specific point's precise location on a grid. This grid is referred to as a coordinate plane.

The distance between two points A(x₁, y₁) and B(x₂, y₂) is given by

[tex]AB = \sqrt{(x_{1} , x_{2})^{2} + (y_{1} - y_{2})^{2} }[/tex]

Observe that the x-coordinate of B is subtracted from the x-coordinate of A. This goes with the y-coordinates.

Therefore, the error is the substitution of coordinates.

The correct computation is

[tex]AB = \sqrt{(6-1)^{2} + [2 - (-4)]^{2} }[/tex]

[tex]= \sqrt{5^{2} + 6^{2} }[/tex]

[tex]= \sqrt{25 + 36} \\[/tex]

[tex]= \sqrt{61}[/tex]

7.81

Learn more about the coordinates here: brainly.com/question/16634867

#SPJ4

The complete question is as follows:

Describe and correct the error in finding the distance between A(6, 2) and B(1, -4). AB = √[(6 - 2)² + {2 - (-4)}²] = √(4² + 5²) = √(16 + 25) = √41 ≈ 6.4.

smart mugs are the next generations of hot drinks dispensers tha om with built in technology to keep drinks at the perfect temperature for hours on end initial cost of a smart mug was aed 896, because of high demand in market the cost increased by 12% find the new price of the mug



PLS QUICK ITS DUE 1 HOUR!!

Answers

With a [tex]12[/tex]% price rise, the smart mug now costs AED [tex]1003.52[/tex].

Price and sell price: what are they?

The sale price is the price an user pays to purchase a thing or a commodity. It is a cost that is higher than the market cost and also includes a portion of the profit. The cost price refers to the price paid by the seller for the item or service.

How would you define price?

Price is the process of figuring out how much a something or service is worth. Price establishes a customer's cost, although it can or cannot be linked to the price a firm pays to manufacture a good or service.

We need to multiply the initial price by [tex]1.12[/tex] which represents a [tex]12[/tex]% increase in decimal form,

New price [tex]=[/tex] Initial price [tex]*[/tex] (1 [tex]+[/tex] Percent increase in decimal form)

New price[tex]= 896 * (1 + 0.12)[/tex]

New price [tex]= 896 * 1.12[/tex]

New price [tex]= 1003.52[/tex]

Therefore, the new price of the smart mug is AED [tex]1003.52[/tex] after a [tex]12[/tex]% increase.

To know more about price visit:

https://brainly.com/question/19091385

#SPJ1

Other Questions
PLEASE HELP ME!i have no clue what to do How did Eugene Talmadge try to block Georgia from implementing the New Deal? GoPro is a company that makes high-definition waterproof cameras. Their primarycommunication strategy is letting users provide some of the content and dominatediscussions. In this case, sharing media means GoPro is _____________. TRUE/FALSE.A torpedo level is a short level that uses an air bubble located inside a liquid filled vial to establish plumb and level references. what happened to the e. coli cells you spread on the lb/ampicillin plates that did not contain a pclone plasmid? What method could have been used to collect the data Which of the following abbreviations does NOT refer to a type of diabetes?NIDDMGDMDICGMDM An exception handler is a piece of code that is written using the try/except statement.true or false what are the s.p.i.c.e outcomes at the end of the Japanese internment? Your organization is considering virtualization solutions. Management wants to ensure that any solution provides the best ROI. Which of the following situations indicates that virtualization would provide the best ROI?a. Most desktop PCs require fast processors and a high amount of memory.b. Most physical servers within the organization are currently underutilized.c. Most physical servers within the organization are currently utilized at close to 100 percent.d. The organization has many servers that do not require failover services. Scenario #3:Imagine you find a map with a scale of 1:63,360. On that map, you see your hiking destination isseven inches from your current location.(a) How far away is that in reality (in miles)?(b) Explain how you arrived at this decision.SHOW YOUR WORK! This includes the potential for partial value, if incorrect. what is carbon dioxide and what causes it to be released into the atmosphere, contributing to global warming? Lesson 4 Invertebrate Evolution and Diversity 1I need help with this lesson if anyone could help me today Which statement below correctly describes their relative atomic radii and first ionization energy when comparing Se and Br? The atomic radius for Se is larger than Br, and the first ionization energy for Se is greater than Br. The atomic radius for Br is larger than Se, and the first ionization energy for Bris greater than Se. The atomic radius for Se is larger than Br, and the first ionization energy for Br is greater than Se. The atomic radius for Br is larger than Se, and the first ionization energy for Se is greater than Br. 8) An advantage of a pure project where self-contained teams work full time on a project is when the following? A. Team members can work on several projects B. Functional area is a "home" after the project is completed C. There are duplicated resources D. Lines of communication are shortened E. Overall organizational policies and goals can be ignored windows subsystem for linux has no installed distributions is called 4. Investigate: In the DESIGN tab, remove the cartilage from one of your designed arms. Compare thenumber of repetitions this arm can do compared to the arm with cartilage match the definitions with their correct term or bond feature. - the rate of interest paid on the face value (par value) of the bond. - the bondholder's required rate of return. this is also the internal rate of return on the cash flows received by the bondholder that assumes the investor can reinvest the cash flows received from the bond at this rate. - the period of time over which the bond issuer must continue to pay coupon payments. - the legal document that states the bond contract requirements such as repayment schedules, restrictions, and promises between the issuer of a corporate bond and the purchaser. Fill in the missing values to make the equations true. 5. In the diagram below, Aircraft A is flying East and maintaining a groundspeed of 340 kt (a kt = speed of 1 NM / hr). Aircraft B is flying in the same direction as aircraft A but 210 NM ahead, maintaining a ground speed of 280 kt. Aircraft A will catch Aircraft B at Point X. What distance will Aircraft B have travelled when this event occurs?